Đến nội dung

Hình ảnh

$ (a+b-c-1)(b+c-a-1)(c+a-b-1) \le 8 $

* * * * - 6 Bình chọn

  • Please log in to reply
Chủ đề này có 45 trả lời

#21
Rias Gremory

Rias Gremory

    Del Name

  • Thành viên
  • 1384 Bài viết

Bài 6

    Cho các số dương a,b,c thỏa mãn $abc=1$.CMR

                    $\frac{1}{a+\sqrt{3a+1}}+\frac{1}{b+\sqrt{3b+1}}+\frac{1}{c+\sqrt{3c+1}}\leq 1$

Bài 7

    Chứng minh rằng với mọi số thực $a,b,c$,ta có

                    $(a^{2}+b^{2})^{2}\geq (a+b-c)(b+c-a)(c+a-b)(a+b+c)$

Bài 8

     Cho $a,b,c>0$ và $a+b+c=3$.CMR với mọi $k>0$,ta có

                     $(b+c)\sqrt[k]{\frac{bc+1}{a^{2}+1}}+(c+a)\sqrt[k]{\frac{ca+1}{b^{2}+1}}+(a+b)\sqrt[k]{\frac{ab+1}{c^{2}+1}}\geq 6$   

Bài 7, 

VT thiếu $c^{2}$ chứ. Vì VP là a,b,c bình đẳng, sao VT chỉ có a và b??



#22
nguyenqn1998

nguyenqn1998

    Trung sĩ

  • Thành viên
  • 173 Bài viết

Bài 14:

Cách 1 trâu bò 

cách 2 ngắn gọn 

ta c/m bất đẳng thức sau $\sqrt{1+x^3}\leq 1+\frac{x^2}{2}$

thật vậy $\sqrt{1+x^3}=\sqrt{(1+x)(1-x+x^2)}\leq \frac{1+x+1-x+x^2}{2}=1+\frac{x^2}{2}$

Áp dụng: 

$\sqrt{\frac{a^3}{a^3+(b+c)^3}}=\frac{1}{\sqrt{(1+(\frac{b+c}{a})^3)}}\geq \frac{1}{1+\frac{1}{2}(\frac{b+c}{a})^2}\geq \frac{1}{1+\frac{b^2+c^2}{a^2}}=\frac{a^2}{a^2+b^2+c^2}$

=> dpcm


Bài viết đã được chỉnh sửa nội dung bởi nguyenqn1998: 06-10-2013 - 12:01


#23
nguyenqn1998

nguyenqn1998

    Trung sĩ

  • Thành viên
  • 173 Bài viết

bạn Nguyên nên đánh dấu số bài.

 

 

Bài 14: Cho ba số a,b,c không âm.CMR

 

$\sum \sqrt{\frac{a^{3}}{a^{3}+\left ( b+c \right )^{3}}}\geq 1$

Bài 14:

Cách 1 trâu bò 

cách 2 ngắn gọn 

ta c/m bất đẳng thức sau $\sqrt{1+x^3}\leq 1+\frac{x^2}{2}$

thật vậy $\sqrt{1+x^3}=\sqrt{(1+x)(1-x+x^2)}\leq \frac{1+x+1-x+x^2}{2}=1+\frac{x^2}{2}$

Áp dụng: 

$\sqrt{\frac{a^3}{a^3+(b+c)^3}}=\frac{1}{\sqrt{(1+(\frac{b+c}{a})^3)}}\geq \frac{1}{1+\frac{1}{2}(\frac{b+c}{a})^2}\geq \frac{1}{1+\frac{b^2+c^2}{a^2}}=\frac{a^2}{a^2+b^2+c^2}$

=> dpcm



#24
LNH

LNH

    Bất Thế Tà Vương

  • Hiệp sỹ
  • 581 Bài viết

Bài 15: $a,b,c>0$. Chứng minh rằng:

$\frac{a}{b}+\frac{b}{c}+\frac{c}{a}\geq \sum \sqrt{\frac{a^2+1}{b^2+1}}$



#25
thuan192

thuan192

    Sĩ quan

  • Thành viên
  • 325 Bài viết

Bài 16: cmr   $\left ( a-\frac{1}{b} \right )\left ( b-\frac{1}{c} \right )\left ( c-\frac{1}{a} \right )\geq \left ( a-\frac{1}{a} \right )\left ( b-\frac{1}{b} \right )\left ( c-\frac{1}{c} \right )$

                  trong đó a,b,c là các số thực không nhỏ hơn 1


:lol:Thuận :lol:

#26
thuan192

thuan192

    Sĩ quan

  • Thành viên
  • 325 Bài viết

BÀI 17: cho các số thực không âm $a_{1},a_{2},a_{3},a_{4},a_{5}$ có tổng bằng 1.   cmr $a_{2}a_{3}a_{4}a_{5}+a_{1}a_{3}a_{4}a_{5}+a_{1}a_{2}a_{3}a_{5}+a_{1}a_{2}a_{3}a_{4}\leq \frac{1}{256}+\frac{3275}{256}a_{1}a_{2}a_{3}a_{4}a_{5}$

 Đẳng thức xảy ra khi nào?


:lol:Thuận :lol:

#27
unvhoang1998

unvhoang1998

    Hạ sĩ

  • Thành viên
  • 58 Bài viết

Góp cho vui nhá!!!

Bài 18: cho $a,b,c\in[1;2]$ thoả $a+b+c =4$

 Tìm MIN: P=$ab+bc+ca+abc$


Bài viết đã được chỉnh sửa nội dung bởi unvhoang1998: 13-10-2013 - 22:15

$\sqrt{\tilde{\mho}}$

 

H$\sigma$$\grave{\alpha}$$\eta$$\varrho$

Không có gì là không thể......... trừ khi bạn không đử dũng khí để tiếp tục làm!!!!

:ukliam2:  :ukliam2:  :ukliam2:  :ukliam2:  :ukliam2:  :ukliam2:  :ukliam2:  :ukliam2:  :ukliam2:  :ukliam2:  :ukliam2:  :ukliam2:  :ukliam2:  :ukliam2:  :ukliam2:  :ukliam2:  :ukliam2:  :ukliam2:  :ukliam2:  :ukliam2:  :ukliam2:

 Rất mong làm quen  MY FACEBOOK


#28
unvhoang1998

unvhoang1998

    Hạ sĩ

  • Thành viên
  • 58 Bài viết

sao chưa ai chém bài mình zậy đành tự xử thôi!!!!!!!!!!!

 từ giả thiết ta có thể đặt: $x=a-1;y=b-1;z=c-1$

khi đó ta được $x,y,z\in[0;1]$ và $x+y+z=1$

nhờ vậy ta được 

$P=(x+1)(y+1)+(y+1)(z+1)+(z+1)(x+1)+(x+1)(y+1)(z+1)=2(xy+yz+zx)+xyz+3(x+y+z)+4$

 $=2(xy+yz+zx)+xyz+7\geq 7$

vậy $MIN P=7$

dấu '=' xảy ra khi $xy+yz+zx=0;xyz=0;x+y+z=1$ hay một trong ba số $a,b,c$ bằng 2 và hai số còn lại bằng 1


$\sqrt{\tilde{\mho}}$

 

H$\sigma$$\grave{\alpha}$$\eta$$\varrho$

Không có gì là không thể......... trừ khi bạn không đử dũng khí để tiếp tục làm!!!!

:ukliam2:  :ukliam2:  :ukliam2:  :ukliam2:  :ukliam2:  :ukliam2:  :ukliam2:  :ukliam2:  :ukliam2:  :ukliam2:  :ukliam2:  :ukliam2:  :ukliam2:  :ukliam2:  :ukliam2:  :ukliam2:  :ukliam2:  :ukliam2:  :ukliam2:  :ukliam2:  :ukliam2:

 Rất mong làm quen  MY FACEBOOK


#29
thuan192

thuan192

    Sĩ quan

  • Thành viên
  • 325 Bài viết

Bài 19: Cho các số thực dương a,b,c thỏa mãn điều kiện 4(a+b+c)-9=0.Tìm GTLN của biểu thức :

                 $S=\left ( a+\sqrt{a^{2}+1} \right )^{b}\left ( b+\sqrt{b^{2}+1} \right )^{c}\left ( c+\sqrt{c^{2}+1} \right )^{a}$


:lol:Thuận :lol:

#30
nguyenqn1998

nguyenqn1998

    Trung sĩ

  • Thành viên
  • 173 Bài viết

Chào các bạn, BĐT là một mảng đề tài rất hay và được nhiều người tìm hiểu.Mình cũng muốn tìm hiẻu thêm về BĐT.

Chúng ta bắt đầu bài đầu tiên:

Bài tâp1:Chứng minh rằng:       $$ (a+b-c-1)(b+c-a-1)(c+a-b-1) \le 8 $$

trong đó $a,b,c$ là các số thực dương thỏa mãn:$ ab+bc+ca=abc$

đặt 2x^3=a+b-c-1

2y^3=b+c-a-1

2z^3=c+a-b-1 (dễ dàng kiểm chứng được phép đặt này đúng) 

là ra ngay



#31
thuan192

thuan192

    Sĩ quan

  • Thành viên
  • 325 Bài viết


Bài3 Cho m là số nguyên lớn hơn 4.Tìm GTLN vàGTNN của biểu thức 

$ab^{m-1}+a^{m-1}b$     trong đó a,b là các số thoả mãn a+b=1 và$0\leq a,b\leq \frac{m-2}{m}$

Đã lâu chưa ai chém mình chém vậy.

         Ta có T=$ab^{m-1}+a^{m-1}b=a\left ( 1-a \right )^{m-1}+a^{m-1}\left ( 1-a \right )=a^{m-1}-a^{m}+\left ( 1-a \right )^{m-1}-\left ( 1-a \right )^{m}$    với $\frac{2}{m}\leq a\leq \frac{m-2}{m}$

   Xét hàm số $f\left ( x \right )=x^{m-1}-x^{m}+\left ( 1-x \right )^{m-1}-\left ( 1-x \right )^{m}$       trên$\left [ \frac{2}{m},\frac{m-2}{m} \right ]$

         Ta có : ${f}'\left ( x \right )=\left ( m-1 \right )x^{m-2}-mx^{m-1}-\left ( m-1 \right )\left ( 1-x \right )^{m-2}+m\left ( 1-x \right )^{m-1}$   (1)

         ${f}''\left ( x \right )=m\left ( m-1 \right )x^{n-3}\left [ \frac{m-2}{m}-x \right ]+m\left ( m-1 \right )\left ( 1-x \right )^{m-3}\left [ x- \frac{2}{m}\right ]$

        Vì  $x\in \left [ \frac{2}{m},\frac{m-2}{m} \right ]$  nên ${f}''\left ( x \right )\geq 0$ do đó ${f}'\left ( x \right )$ đồng biến .

         Với $\frac{1}{2}< x\leq \frac{m-2}{m}$ =>${f}'\left ( x \right )\leq > {f}'\left ( \frac{1}{2} \right )=0$

      => $f\left ( x \right )$ đồng biến trên $\left [ \frac{1}{2},\frac{m-2}{m} \right ]$

         với$\frac{2}{m}< x< \frac{1}{2}$ => ${f}'\left ( x \right )< {f}'\left ( \frac{1}{2} \right )=0$ =>$f\left ( x \right )$ nghịch biến trên $\left [ \frac{2}{m},\frac{1}{2} \right ]$

                Do đó $f_{min}=f\left ( \frac{1}{2} \right )=2^{1-m}$

                               $f_{max}=max\left \{ f\left ( \frac{2}{m} \right ),f\left ( \frac{m-2}{m} \right ) \right \}=\left ( \frac{m-2}{m} \right )\left ( \frac{2}{m} \right )^{m-1}+\left ( \frac{2}{m} \right )\left ( \frac{m-2}{m} \right )^{m-1}$


:lol:Thuận :lol:

#32
nguyenqn1998

nguyenqn1998

    Trung sĩ

  • Thành viên
  • 173 Bài viết

Đã lâu chưa ai chém mình chém vậy.

         Ta có T=$ab^{m-1}+a^{m-1}b=a\left ( 1-a \right )^{m-1}+a^{m-1}\left ( 1-a \right )=a^{m-1}-a^{m}+\left ( 1-a \right )^{m-1}-\left ( 1-a \right )^{m}$    với $\frac{2}{m}\leq a\leq \frac{m-2}{m}$

   Xét hàm số $f\left ( x \right )=x^{m-1}-x^{m}+\left ( 1-x \right )^{m-1}-\left ( 1-x \right )^{m}$       trên$\left [ \frac{2}{m},\frac{m-2}{m} \right ]$

         Ta có : ${f}'\left ( x \right )=\left ( m-1 \right )x^{m-2}-mx^{m-1}-\left ( m-1 \right )\left ( 1-x \right )^{m-2}+m\left ( 1-x \right )^{m-1}$   (1)

         ${f}''\left ( x \right )=m\left ( m-1 \right )x^{n-3}\left [ \frac{m-2}{m}-x \right ]+m\left ( m-1 \right )\left ( 1-x \right )^{m-3}\left [ x- \frac{2}{m}\right ]$

        Vì  $x\in \left [ \frac{2}{m},\frac{m-2}{m} \right ]$  nên ${f}''\left ( x \right )\geq 0$ do đó ${f}'\left ( x \right )$ đồng biến .

         Với $\frac{1}{2}< x\leq \frac{m-2}{m}$ =>${f}'\left ( x \right )\leq > {f}'\left ( \frac{1}{2} \right )=0$

      => $f\left ( x \right )$ đồng biến trên $\left [ \frac{1}{2},\frac{m-2}{m} \right ]$

         với$\frac{2}{m}< x< \frac{1}{2}$ => ${f}'\left ( x \right )< {f}'\left ( \frac{1}{2} \right )=0$ =>$f\left ( x \right )$ nghịch biến trên $\left [ \frac{2}{m},\frac{1}{2} \right ]$

                Do đó $f_{min}=f\left ( \frac{1}{2} \right )=2^{1-m}$

                               $f_{max}=max\left \{ f\left ( \frac{2}{m} \right ),f\left ( \frac{m-2}{m} \right ) \right \}=\left ( \frac{m-2}{m} \right )\left ( \frac{2}{m} \right )^{m-1}+\left ( \frac{2}{m} \right )\left ( \frac{m-2}{m} \right )^{m-1}$

bài này thiên về ĐH hơn là olympic vì vậy mong bạn đừng có đăng những bài như thế này!!  :)



#33
Yagami Raito

Yagami Raito

    Master Tetsuya

  • Thành viên
  • 1333 Bài viết

đặt 2x^3=a+b-c-1

2y^3=b+c-a-1

2z^3=c+a-b-1 (dễ dàng kiểm chứng được phép đặt này đúng) 

là ra ngay

Anh giải bài 1 cụ thể hơn được không ? 


:nav: Học gõ công thức toán học tại đây

:nav: Hướng dẫn đặt tiêu đề tại đây

:nav: Hướng dẫn Vẽ hình trên diễn đàn toán tại đây

--------------------------------------------------------------

 


#34
thuan192

thuan192

    Sĩ quan

  • Thành viên
  • 325 Bài viết

đặt 2x^3=a+b-c-1

2y^3=b+c-a-1

2z^3=c+a-b-1 (dễ dàng kiểm chứng được phép đặt này đúng) 

là ra ngay

mình xin giải tiếp như sau:

  Từ ab+bc+ca=abc và a,b,c>0 nên ta có abc lớn hơn mỗi tích ab,bc,ca và do đó ta có a,b,c >1

   Giả sử xyz>1 thì do a,b,c>1 nên tổng hai trong ba số $x^{3},y^{3},z^{3}$ là dương suy ra có tối đa một số trong x,y,z là âm . Do xyz>1 nên cả ba số x,y,z đều dương . Từ đó ta có 

                 $y^{3}+z^{3}=\left ( y+z \right )\left [ yz+\left ( y-z \right )^{2} \right ]\geq yz\left ( y+z \right )$

 suy ra: $x\left ( y^{3}+z^{3}+1 \right )\geq xyz\left ( y+z \right )+x> x+y+z$

suy ra: $\frac{x}{x+y+z}> \frac{1}{y^{3}+z^{3}+1}=\frac{1}{a}$

 tương tự cho y,z rồi cộng các bđt lại với nhau suy ra abc>a+b+c (mâu thuẫn)

 suy ra xyz<1


:lol:Thuận :lol:

#35
thanhdotk14

thanhdotk14

    Thượng sĩ

  • Thành viên
  • 268 Bài viết

Góp vui một bài, bài này khá độc đáo =))

________________________________

Bài 20: Cho các số thực $a_1,a_2,a_3,a_4,a_5\in [0;4]$ và thỏa mãn:$$a_1+a_2+a_3+a_4+a_5=10$$

Tìm $\max$ của biểu thức:$$P=a_1^3+a_2^3+a_3^3+a_4^3+a_5^3$$


-----------------------------------------------------

 

:ukliam2: Untitled1_zps6cf4d69d.jpg :ukliam2:


#36
nguyenqn1998

nguyenqn1998

    Trung sĩ

  • Thành viên
  • 173 Bài viết


Góp vui một bài, bài này khá độc đáo =))

________________________________

Bài 20: Cho các số thực $a_1,a_2,a_3,a_4,a_5\in [0;4]$ và thỏa mãn:$$a_1+a_2+a_3+a_4+a_5=10$$

Tìm $\max$ của biểu thức:$$P=a_1^3+a_2^3+a_3^3+a_4^3+a_5^3$

đặt $f(x)=x^3$ => $f'(x)=6x \geq 0$

mà bộ $(4,4,2,0,0)$ trội hơn bộ $(a_1,a_2,a_3,a_4,a_5)$ (hiển nhiên)

nên $f(4)+f(4)+f(2)+f(0)+f(0)\geq f(a_1)+f(a_2)+f(a_3)+f(a_4)+f(a_5)$ (theo bất đẳng thức karamata)

<=> $136 \geq a_1^3+a_2^3+a_3^3+a_4^3+a_5^3$

dấu '=' xảy ra khi a1=a2=4, a3=2,a4=a5=0 và các hoán vị của nó


Bài viết đã được chỉnh sửa nội dung bởi nguyenqn1998: 06-11-2013 - 22:32


#37
thanhdotk14

thanhdotk14

    Thượng sĩ

  • Thành viên
  • 268 Bài viết

đặt $f(x)=x^3$ => $f'(x)=6x \geq 0$

mà bộ $(4,4,2,0,0)$ trội hơn bộ $(a_1,a_2,a_3,a_4,a_5)$ (hiển nhiên)

nên $f(4)+f(4)+f(2)+f(0)+f(0)\geq f(a_1)+f(a_2)+f(a_3)+f(a_4)+f(a_5)$ (theo bất đẳng thức karamata)

<=> $136 \geq a_1^3+a_2^3+a_3^3+a_4^3+a_5^3$

dấu '=' xảy ra khi a1=a2=4, a3=2,a4=a5=0 và các hoán vị của nó

Đề nghị em chứng minh rõ rành nha  :icon6:  :icon6:  :icon6:

_______________________________________

Bài giải:

Không mất tính tổng quát, ta có thể giả sử:$4\ge a_1\ge a_2\ge a_3\ge a_4\ge a_5\ge 0$

Từ đó suy ra:$$\left\{\begin{matrix} a_1\le 4 & & \\ a_1+a_2\le 8 & & \\ a_1+a_2+a_3\le 10 & & \\ a_1+a_2+a_3+a_4\le 10 & & \\ a_1+a_2+a_3+a_4+a_5=10 & & \end{matrix}\right.$$

Ta có:$$P=a_1^3+a_2^3+a_3^3+a_4^3+a_5^3$$

$$=a_1(a_1^2-a_2^2)+(a_1+a_2)(a_2^2-a_3^2)+(a_1+a_2+a_3)(a_3^2-a_4^2)+(a_1+a_2+a_3+a_4)(a_4^2-a_5^2)+(a_1+a_2+a_3+a_4+a_5)a_5^2$$

$$\le 4(a_1^2-a_2^2)+8(a_2^2-a_3^2)+10(a_3^2-a_4^2)+10(a_4^2-a_5^2)+10a_5^2$$

$$=2(2a_1^2+2a_2^2+a_3^2)$$

Lại có:$$a_1^2+a_2^2+a_3^2=a_1(a_1-a_2)+(a_1+a_2)(a_2-a_3)+(a_1+a_2+a_3)a_3$$

$$\le 4(a_1+a_2)+2a_3=2(a_1+a_2)+2(a_1+a_2+a_3)\le 36$$

Từ đó suy ra:$$P\le 2(36+32)=136$$

Đẳng thức xảy ra $\Leftrightarrow a_1=a_2=4,a_3=2,a_4=a_5=0$ và các hoán vị


Bài viết đã được chỉnh sửa nội dung bởi thanhdotk14: 07-11-2013 - 18:33

-----------------------------------------------------

 

:ukliam2: Untitled1_zps6cf4d69d.jpg :ukliam2:


#38
thuan192

thuan192

    Sĩ quan

  • Thành viên
  • 325 Bài viết

Bài 2

Cho $a_{1},a_{2},...,a_{k}$ là các số thực dương thỏa $a_{1}a_{2}...a_{k}=\alpha$. Chứng minh rằng:

$a_{1}+a_{2}+...+a_{k}\geq \alpha _{1}p_{1}+\alpha _{2}p_{2}+...+\alpha _{j}p_{j}$

Biết rằng $\alpha =p_{1}^{\alpha _{1}}p_{2}^{\alpha _{2}}...p_{j}^{\alpha_{j}}$ và $p_{i}$ là các số nguyên tố

Nhận xét: Nếu $a,b\geq 2$ thì $ab\geq a+b$

Thật vậy:  $ab\geq a+b$ $\Rightarrow 2ab\geq 2a+2b\Rightarrow a\left ( b-2 \right )+b\left ( a-2 \right )$ (đúng)

Nếu trong tích $a_{1}a_{2}...a_{n}=\alpha$ có một thữa số $a_{j}=4$ thì ta thay $a_{j}=2.2$ .Lúc đó ta có:

$\left\{\begin{matrix} a_{1}a_{2}...a_{k}=a_{1}a_{2}...a_{j-1}.2.2.a_{j+1}...a_{k}=\alpha \\a_{1}+a_{2}+...+a_{k}=a_{1}+a_{2}+...+a_{j-1}+2+2+a_{j+1}+...+a_{k} \end{matrix}\right.$

(nghĩa là trong trường hợp này tích và tổng $a_{1},a_{2},...a_{k}$$a_{1},a_{2},...a_{k}$ có một thừa số $a_{j}$ là hợp số >4.Thay $a_{j}$ bằng phân tích ra thừ số nguyên tố của nó,chẳng hạn $a_{j}=q_{1}^{\beta _{1}}q_{2}^{^{\beta _{_{2}}}}...q_{s}^{\beta _{s}}$ ,ở đây $q_{1},q_{2},...,q_{s}$ là các số nguyên tố (dĩ nhiên $\geq 2$ và $\beta _{1},\beta _{2},...\beta _{s}$ là các số nguyên dương.Theo nhận xét trên (áp dụng nhiều lần; mỗi lần với một thừa số một),ta có:

   $a_{j}\geq q_{1}+...+q_{1}+....+q_{s}+...+q_{s}=\beta _{1}q_{1}+...+\beta _{s}q_{s}$

mặt khác $a_{1}a_{2}...s_{j}...a_{k}=a_{1}a_{2}...a_{j-1}\left ( q_{1}^{\beta _{1}}...q_{s}^{\beta _{s}} \right )a_{j+1}...a_{k}=\alpha$

Lần lượt thay $a_{1},a_{2},...,a_{k}$ bằng các phân tích ra thừa số nguyên tố của nó ta được đpcm


:lol:Thuận :lol:

#39
thuan192

thuan192

    Sĩ quan

  • Thành viên
  • 325 Bài viết

Bài 21: Giả sử a,b,c là ba cạnh của một tam giác .cmr $\sum \frac{a}{\sqrt{a^{2}+3bc}}\geq \frac{3}{2}$


  • LNH yêu thích
:lol:Thuận :lol:

#40
nhatquangsin

nhatquangsin

    Thượng sĩ

  • Thành viên
  • 238 Bài viết

Bài 21: Giả sử a,b,c là ba cạnh của một tam giác .cmr $\sum \frac{a}{\sqrt{a^{2}+3bc}}\geq \frac{3}{2}$

 

Do bất đẳng thức thuần nhất bậc 0 nên ta chuẩn hoá $a+b+c=3$

Ta có $\sum _{cyc}\frac{a}{\sqrt{a^2+3bc}}\geq \frac{9}{\sum a\sqrt{a^2+3bc}}\geq \frac{36}{\sum 5a^2+3bc}=\frac{36}{5(a+b+c)^2-7(ab+bc+ca)}\geq \frac{3}{2}$

(Do $ab+bc+ca\leq3$)

sorry các bạn, cách giải này có 1 lỗi

----------------------------------------

Bài 22: Cho $a_1,a_2,...,a_n$ là các số dương. Chứng minh rằng

$(1+\frac{a_1^2}{a_2})(1+\frac{a_2^2}{a_3})...(1+\frac{a_n^2}{a_1})\leq (1+\frac{a_1^3}{a_2a_3})(1+\frac{a_2^3}{a_3a_4})...(1+\frac{a_n^3}{a_1a_2})$


Bài viết đã được chỉnh sửa nội dung bởi nhatquangsin: 20-11-2013 - 18:56

  • LNH yêu thích




1 người đang xem chủ đề

0 thành viên, 1 khách, 0 thành viên ẩn danh